www.vorhilfe.de
- Förderverein -
Der Förderverein.

Gemeinnütziger Verein zur Finanzierung des Projekts Vorhilfe.de.
Hallo Gast!einloggen | registrieren ]
Startseite · Mitglieder · Impressum
Forenbaum
^ Forenbaum
Status VH e.V.
  Status Vereinsforum

Gezeigt werden alle Foren bis zur Tiefe 2

Navigation
 Startseite...
 Suchen
 Impressum
Das Projekt
Server und Internetanbindung werden durch Spenden finanziert.
Organisiert wird das Projekt von unserem Koordinatorenteam.
Hunderte Mitglieder helfen ehrenamtlich in unseren moderierten Foren.
Anbieter der Seite ist der gemeinnützige Verein "Vorhilfe.de e.V.".
Partnerseiten
Weitere Fächer:

Open Source FunktionenplotterFunkyPlot: Kostenloser und quelloffener Funktionenplotter für Linux und andere Betriebssysteme
Forum "Wahrscheinlichkeitstheorie" - fast sichere Konvergenz
fast sichere Konvergenz < Wahrscheinlichkeitstheorie < Stochastik < Hochschule < Mathe < Vorhilfe
Ansicht: [ geschachtelt ] | ^ Forum "Wahrscheinlichkeitstheorie"  | ^^ Alle Foren  | ^ Forenbaum  | Materialien

fast sichere Konvergenz: Frage (beantwortet)
Status: (Frage) beantwortet Status 
Datum: 18:35 So 16.03.2014
Autor: couldbeworse

Aufgabe
Es seien [mm] $X_1, X_2,...$ [/mm] unabhängige Zufallsvariablen mit identischer Verteilung. Die Zufallsvariablen seien nichtnegativ und es existiere [mm] $E[X_1^k]$ [/mm] für alle [mm] $k\in\IN$. [/mm]

a) Zeigen Sie, dass [mm] $\frac{1}{\sqrt{n}}X_n$ [/mm] fast sicher gegen 0 konvergiert.

b) Bestimmen Sie [mm] $\lim_{n\to\infty} [/mm] P [mm] \left( \sum_{k=1}^n (X_k - E[X_k]) \le X_n\right)$. [/mm]

Hallo zusammen!

Dies ist eine alte Klausuraufgabe, kann also eigentlich gar nicht so schwer sein aber mit Konvergenz von Zufallsvariablen stehe ich leider auf Kriegsfuß.


Beim Teil a) ist als Hinweis noch angegeben, dass man das Lemma von Borel-Cantelli verwenden soll. Ich muß ja [mm] $P\left(\lim_{n\to\infty} \frac{1}{\sqrt{n}}X_n=0\right)=1$ [/mm] zeigen. Angenommen das wäre nicht der Fall, dann müsste es ein [mm] $\epsilon [/mm] >0$ geben, sodass unendlich oft [mm] $|\frac{1}{\sqrt{n}}X_n [/mm] -0|= [mm] \frac{1}{\sqrt{n}}X_n >\epsilon$ [/mm] gilt. Diese Ereignisse sollen die [mm] $A_n$ [/mm] sein. Wenn [mm] $\sum_{n\ge 1}^{\infty}P(A_n)<\infty$ [/mm] ist, dann folgt doch aus dem Lemma von Borel-Cantelli [mm] $P(\limsup A_n)=0$ [/mm] und damit die Behauptung, oder? Nur krieg ich das leider nicht hin.


Bei der b) dachte ich, dass nach dem schwachen Gesetz der großen Zahlen gilt: [mm] $\lim_{n\to\infty}P\left(|\frac{1}{n}\sum_{k=1}^n X_k - E[X_1]>\epsilon\right)=0$. [/mm]

Also [mm] $\lim_{n\to\infty}P \left( \sum_{k=1}^n (X_k - E[X_k]) \le X_n\right)=\lim_{n\to\infty}P \left( \sum_{k=1}^n (X_k - E[X_1]) \le X_n\right) [/mm] = [mm] \lim_{n\to\infty}P \left( \frac{1}{n}\sum_{k=1}^n X_k - E[X_1] \le \frac{1}{n}X_n\right)= [/mm] 1 - [mm] \lim_{n\to\infty}P \left( \frac{1}{n}\sum_{k=1}^n X_k - E[X_1] > \frac{1}{n}X_n\right)= [/mm] 1-0=1.$ Stimmt das so?

Vielen Dank für eure Hilfe!

Gruß couldbeworse

Ich habe diese Frage in keinen anderen Internetforen gestellt.

        
Bezug
fast sichere Konvergenz: Antwort
Status: (Antwort) fertig Status 
Datum: 11:18 Mo 17.03.2014
Autor: tobit09

Hallo couldbeworse!


> Es seien [mm]X_1, X_2,...[/mm] unabhängige Zufallsvariablen mit
> identischer Verteilung. Die Zufallsvariablen seien
> nichtnegativ und es existiere [mm]E[X_1^k][/mm] für alle [mm]k\in\IN[/mm].
>  
> a) Zeigen Sie, dass [mm]\frac{1}{\sqrt{n}}X_n[/mm] fast sicher gegen
> 0 konvergiert.
>
> b) Bestimmen Sie [mm]\lim_{n\to\infty} P \left( \sum_{k=1}^n (X_k - E[X_k]) \le X_n\right)[/mm].


> Dies ist eine alte Klausuraufgabe, kann also eigentlich gar
> nicht so schwer sein aber mit Konvergenz von
> Zufallsvariablen stehe ich leider auf Kriegsfuß.

Mir erscheint diese Aufgabe für eine Klausuraufgabe eigentlich zu schwer. Oder ich habe leichtere Lösungswege übersehen... ;-)


> Beim Teil a) ist als Hinweis noch angegeben, dass man das
> Lemma von Borel-Cantelli verwenden soll. Ich muß ja
> [mm]P\left(\lim_{n\to\infty} \frac{1}{\sqrt{n}}X_n=0\right)=1[/mm]
> zeigen.

Genau.

> Angenommen das wäre nicht der Fall, dann müsste
> es ein [mm]\epsilon >0[/mm] geben, sodass unendlich oft
> [mm]|\frac{1}{\sqrt{n}}X_n -0|= \frac{1}{\sqrt{n}}X_n >\epsilon[/mm]
> gilt.

Das stimmt so nicht. Es würde stimmen, wenn [mm] $X_n$ [/mm] eine Folge reeller Zahlen und keine Folge von Zufallsgrößen wäre.

Aber man kann zeigen, dass im Falle der "Nicht-fast-sicheren-Konvergenz" von [mm] $\frac{1}{\sqrt{n}}X_n$ [/mm] gegen 0 ein [mm] $\varepsilon>0$ [/mm] existieren würde mit

(*)     [mm] $P(\limsup_{n\to\infty}\{\frac{1}{\sqrt{n}}X_n>\varepsilon\})>0$. [/mm]


> Diese Ereignisse sollen die [mm]A_n[/mm] sein. Wenn [mm]\sum_{n\ge 1}^{\infty}P(A_n)<\infty[/mm]
> ist, dann folgt doch aus dem Lemma von Borel-Cantelli
> [mm]P(\limsup A_n)=0[/mm] und damit die Behauptung, oder?

Ja, dann folgt ein Widerspruch zu (*).

> Nur krieg
> ich das leider nicht hin.

Mir fällt leider kein schönerer Weg als folgender ein:

Es gilt wegen [mm] $X_n\ge0$ [/mm]

      [mm] $A_n=\{\frac{X_n^4}{\varepsilon^4}>n^2\}$. [/mm]

Also folgt mit der identischen Verteilung der [mm] $X_n$ [/mm] und der Markov-Ungleichung

     [mm] $P(A_n)=P(\frac{X_1^4}{\varepsilon^4}>n^2)\le \frac{E \frac{(X_1)^4}{\varepsilon^4}}{n^2}$. [/mm]

Kriegst du damit [mm] $\sum_{n=1}^\infty P(A_n)<\infty$ [/mm] gezeigt?


> Bei der b) dachte ich, dass nach dem schwachen Gesetz der
> großen Zahlen

für alle reellen Zahlen [mm] $\varepsilon>0$ [/mm]

> gilt:
> [mm]\lim_{n\to\infty}P\left(|\frac{1}{n}\sum_{k=1}^n X_k - E[X_1]>\epsilon\right)=0[/mm].
>
> Also [mm]\lim_{n\to\infty}P \left( \sum_{k=1}^n (X_k - E[X_k]) \le X_n\right)=\lim_{n\to\infty}P \left( \sum_{k=1}^n (X_k - E[X_1]) \le X_n\right) = \lim_{n\to\infty}P \left( \frac{1}{n}\sum_{k=1}^n X_k - E[X_1] \le \frac{1}{n}X_n\right)= 1 - \lim_{n\to\infty}P \left( \frac{1}{n}\sum_{k=1}^n X_k - E[X_1] > \frac{1}{n}X_n\right)= 1-0=1.[/mm]
> Stimmt das so?

Nein, der vorletzte Schritt ist falsch. [mm] $\frac{1}{n}X_n$ [/mm] ist keine feste reelle Zahl, sondern eine Zufallsvariable (die zu allem Unglück auch noch von $n$ abhängt...). Also kannst du nicht [mm] $\varepsilon:=\frac{1}{n}X_n$ [/mm] wählen.


Die Idee bei b) ist, den zentralen Grenzwertsatz und a) (oder zumindest die stochastische Konvergenz von [mm] $\frac{1}{\sqrt{n}}X_n$ [/mm] gegen 0) ins Spiel zu bringen:

Sei [mm] $\mu:=EX_1$ [/mm] und [mm] $\sigma:=\sqrt{\operatorname{Var}X_1}$. [/mm]

Den Fall [mm] $\sigma=0$ [/mm] (dann sind alle [mm] $X_n$ [/mm] konstant) überlasse ich dir. Sei nun [mm] $\sigma>0$. [/mm]

Wir können die Ereignisse

     [mm] $B_n:=\{\sum_{k=1}^n (X_k-EX_k)\le X_n\}$ [/mm]

aus der Aufgabenstellung mit [mm] $\mu:=EX_1$ [/mm] und [mm] $\sigma^2:=\operatorname{Var}X_1$ [/mm] umschreiben zu

     [mm] $B_n=\{\underbrace{\frac{1}{\sqrt{n}*\sigma}(\sum_{k=1}^nX_k-n*\mu)}_{=:Z_n}\le\frac{1}{\sigma}*\frac{1}{\sqrt{n}}X_n\}$. [/mm]

(Die linke Seite vom [mm] $\le$-Zeichen [/mm] sieht schon sehr nach dem zentralen Grenzwertsatz aus, aber auf der rechten Seite steht leider noch keine reelle Zahl...)

Für [mm] $\delta>0$ [/mm] sei

     [mm] $C_{n,\delta}:=\{\frac{1}{\sigma}*\frac{1}{\sqrt{n}}X_n>\delta\}$. [/mm]

Wir wollen [mm] $P(B_n)$ [/mm] nach oben und unten abschätzen. Es gilt für alle [mm] $\delta>0$ [/mm]

     [mm] $\{Z_n\le0\}\subseteq B_n=(B_n\cap C_{n,\delta})\cup(B_n\cap (C_{n,\delta})^c)\subseteq C_{n,\delta}\cup\{Z_n\le\delta\}$ [/mm]

und somit

     [mm] $P(\{Z_n\le 0\})\le P(B_n)\le P(C_{n,\delta})+P(Z_n\le\delta)$. [/mm]

Wie verhalten sich die Folgen

     [mm] $(P(\{Z_n\le 0\}))_{n\in\IN}$ [/mm]

und

     [mm] $(P(C_{n,\delta})+P(Z_n\le\delta))_{n\in\IN}$ [/mm]

für $n$ gegen [mm] $\infty$? [/mm] Wende dazu den zentralen Grenzwertsatz und Teil a) an.


Viele Grüße
Tobias

Bezug
Ansicht: [ geschachtelt ] | ^ Forum "Wahrscheinlichkeitstheorie"  | ^^ Alle Foren  | ^ Forenbaum  | Materialien


^ Seitenanfang ^
ev.vorhilfe.de
[ Startseite | Mitglieder | Impressum ]